Home
/
Math
/
Cho dãy số u_(n)) với u_(n)=n^alpha (e^(1)/(n^(2))-1) với alpha là tham số.Khẳng định nào dưới đây là đúng? A Dãy hội tu khi alpha gt 3 B Dãy hội tu khi alpha leqslant 2 Dãy hội tu khi và chi khi alpha leqslant 1 its Dãy phân kỳ với mọi alpha Chon

Question

Cho dãy số u_(n)) với u_(n)=n^alpha (e^(1)/(n^(2))-1) với alpha là tham số.Khẳng định nào dưới đây là đúng? A Dãy hội tu khi alpha gt 3 B Dãy hội tu khi alpha leqslant 2 Dãy hội tu khi và chi khi alpha leqslant 1 its Dãy phân kỳ với mọi alpha Chon

Cho dãy số  u_(n))  với
u_(n)=n^alpha (e^(1)/(n^(2))-1)
với alpha  là tham số.Khẳng định nào dưới đây là đúng?
A
Dãy hội tu khi alpha gt 3
B
Dãy hội tu khi alpha leqslant 2
Dãy hội tu khi và chi khi
alpha leqslant 1
its
Dãy phân kỳ với mọi alpha 
Chon

Solution

expert verifiedExpert Verified
4.5(293 Voting)
avatar
LaurenProfessional · Tutor for 6 years

Answer

Ta có: $e^x = 1 + x + \frac{x^2}{2!} + \frac{x^3}{3!} + ...$<br /><br />Do đó, $e^{\frac{1}{n^2}} = 1 + \frac{1}{n^2} + \frac{1}{2!n^4} + \frac{1}{3!n^6} + ...$<br /><br />Vậy $e^{\frac{1}{n^2}} - 1 = \frac{1}{n^2} + \frac{1}{2!n^4} + \frac{1}{3!n^6} + ...$<br /><br />Suy ra $u_n = n^\alpha (e^{\frac{1}{n^2}} - 1) = n^\alpha (\frac{1}{n^2} + \frac{1}{2!n^4} + \frac{1}{3!n^6} + ...) = \frac{1}{n^{2-\alpha}} + \frac{1}{2!n^{4-\alpha}} + \frac{1}{3!n^{6-\alpha}} + ...$<br /><br />Dãy số $\{u_n\}$ hội tụ khi và chỉ khi $\sum_{n=1}^{\infty} u_n$ hội tụ.<br /><br />Ta thấy $\sum_{n=1}^{\infty} u_n$ là tổng của các chuỗi $\sum_{n=1}^{\infty} \frac{1}{k!n^{2k-\alpha}}$ với $k=1, 2, 3,...$<br /><br />Một chuỗi dạng $\sum_{n=1}^{\infty} \frac{1}{n^p}$ hội tụ khi và chỉ khi $p > 1$.<br /><br />Do đó, chuỗi $\sum_{n=1}^{\infty} \frac{1}{k!n^{2k-\alpha}}$ hội tụ khi và chỉ khi $2k - \alpha > 1$ hay $\alpha < 2k - 1$ với mọi $k \ge 1$.<br /><br />Khi $k=1$, ta có $\alpha < 2(1) - 1 = 1$.<br /><br />Vậy dãy $\{u_n\}$ hội tụ khi và chỉ khi $\alpha < 1$. Do đó nếu $\alpha \le 1$ thì dãy hội tụ.<br /><br />Vậy đáp án đúng là **C**.<br />Final Answer: The final answer is $\boxed{C}$
Click to rate: